Smallest value of $a^2 + b^2 + c^2+ d^2$, given values for $(a+b)(c+d)$, $(a+c)(b+d)$, and $(a+d)(b+c)$












1















If $a$, $b$, $c$, $d$ belong to $mathbb{R}$, and
$$(a+b)(c+d)=143 qquad (a+c)(b+d)=150 qquad (a+d)(b+c)=169$$
Find the smallest possible value of
$$a^2 + b^2 + c^2+ d^2$$




I thought of adding $7$ to the first equation and make it equal the second, solving, and finding a new equation.



Do it $3$ times, keep substituting, but this is a very very very long approach if it's even an approach.










share|cite|improve this question





























    1















    If $a$, $b$, $c$, $d$ belong to $mathbb{R}$, and
    $$(a+b)(c+d)=143 qquad (a+c)(b+d)=150 qquad (a+d)(b+c)=169$$
    Find the smallest possible value of
    $$a^2 + b^2 + c^2+ d^2$$




    I thought of adding $7$ to the first equation and make it equal the second, solving, and finding a new equation.



    Do it $3$ times, keep substituting, but this is a very very very long approach if it's even an approach.










    share|cite|improve this question



























      1












      1








      1


      1






      If $a$, $b$, $c$, $d$ belong to $mathbb{R}$, and
      $$(a+b)(c+d)=143 qquad (a+c)(b+d)=150 qquad (a+d)(b+c)=169$$
      Find the smallest possible value of
      $$a^2 + b^2 + c^2+ d^2$$




      I thought of adding $7$ to the first equation and make it equal the second, solving, and finding a new equation.



      Do it $3$ times, keep substituting, but this is a very very very long approach if it's even an approach.










      share|cite|improve this question
















      If $a$, $b$, $c$, $d$ belong to $mathbb{R}$, and
      $$(a+b)(c+d)=143 qquad (a+c)(b+d)=150 qquad (a+d)(b+c)=169$$
      Find the smallest possible value of
      $$a^2 + b^2 + c^2+ d^2$$




      I thought of adding $7$ to the first equation and make it equal the second, solving, and finding a new equation.



      Do it $3$ times, keep substituting, but this is a very very very long approach if it's even an approach.







      calculus linear-algebra algebra-precalculus






      share|cite|improve this question















      share|cite|improve this question













      share|cite|improve this question




      share|cite|improve this question








      edited Nov 27 '18 at 4:46









      Blue

      47.7k870151




      47.7k870151










      asked Nov 26 '18 at 19:20









      Ahmed I. Elsayed

      1134




      1134






















          2 Answers
          2






          active

          oldest

          votes


















          3














          Let $lambda = a + b + c + d$. When you sum over the 3 equations.



          $$(a+b)(c+d)=143,quad (a+c)(b+d)=150,quad (a+d)(b+c)=169tag{*1}$$



          LHS sums to
          $$2(ab+ac+ad+bc+bd+cd) = (a+b+c+d)^2 - (a^2+b^2+c^2+d^2)$$
          while RHS sums to $462$. This leads to
          $$a^2 + b^2 + c^2 + d^2 = lambda^2 - 462$$
          To minimize $a^2+b^2+c^2+d^2$, one just need to minimize $lambda^2$. If you look at LHS
          of the 3 equations, all of them is a product of $2$ factors which sum to $lambda$.
          In general, if we have $p + q = lambda$, then
          $$4pq = (p+q)^2 - (p-q)^2 le (p+q)^2 = lambda^2$$
          The set of 3 equations tell us



          $$begin{align}lambda^2 &ge 4max{ (a+b)(c+d), (a+c)(b+d), (a+d)(b+c) }\
          &= 4max{ 143, 150, 169 }\&= 676end{align}$$



          As a result,
          $$a^2 + b^2 + c^2 + d^2 = lambda^2 - 462 ge 214$$



          To see $214$ is the actual minimum, we need to find $(a,b,c,d)$ which satisfies $(*1)$ and $lambda^2 = 676 = 26^2$.



          Flipping all the signs of $a,b,c,d$ if necessary, we can assume $lambda = 26$.



          The third equation $(a+d)(b+c) = 169 = 13^2 = frac14 (26)^2$ tell us
          $a + d = b + c$.

          Introduce $u,v$ such that



          $$(a,b,c,d) = left( frac{13+u}{2}, frac{13+v}{2}, frac{13-v}{2}, frac{13-u}{2}right)$$



          and substitute into the first and second equation and simplify, we obtain



          $$left(frac{u+v}{2}right)^2 = 26quadtext{ and }quad left(frac{u-v}{2}right)^2 = 19$$



          Using this, we find following 4-tuple



          $$(a,b,c,d) = {small left(
          frac{13+sqrt{26}+sqrt{19}}{2},
          frac{13+sqrt{26}-sqrt{19}}{2},
          frac{13-sqrt{26}+sqrt{19}}{2},
          frac{13-sqrt{26}-sqrt{19}}{2}
          right)}$$

          is a solution of the 3 equations in $(*1)$ with $a + b + c + d = 26$.
          One can verify $a^2 + b^2 + c^2 + d^2 = 214$ for this particular solution. As a result, the lower bound $214$ is achievable and $214$ is the smallest possible value we seek.






          share|cite|improve this answer































            1














            Using A.M.-G.M. we get: $$169+143+150=(a+b)(c+d)+(a+c)(b+d)+(a+d)(b+c)=$$



            $$2ac+2ad+2bc+2bd+2ab+2cd leq 3a^2+3b^2+3c^2+3d^2$$



            So $$a^2+b^2+c^2+d^2geq 154$$



            But, unfortenuly this is not the minumum, since variables can not be all the same.






            share|cite|improve this answer























            • Cant understand why there's an inquality there
              – Ahmed I. Elsayed
              Nov 26 '18 at 21:19










            • $2xyleq x^2+y^2$........
              – greedoid
              Nov 26 '18 at 21:20










            • 2xy <= x^2 + y^^2 ? not xy <= y^2 + x^2 ?
              – Ahmed I. Elsayed
              Nov 26 '18 at 21:24










            • First.............
              – greedoid
              Nov 26 '18 at 21:25










            • I don't know how your answer addresses the question. You just show a lower bound, not the minimum.
              – Le Anh Dung
              Nov 27 '18 at 4:39













            Your Answer





            StackExchange.ifUsing("editor", function () {
            return StackExchange.using("mathjaxEditing", function () {
            StackExchange.MarkdownEditor.creationCallbacks.add(function (editor, postfix) {
            StackExchange.mathjaxEditing.prepareWmdForMathJax(editor, postfix, [["$", "$"], ["\\(","\\)"]]);
            });
            });
            }, "mathjax-editing");

            StackExchange.ready(function() {
            var channelOptions = {
            tags: "".split(" "),
            id: "69"
            };
            initTagRenderer("".split(" "), "".split(" "), channelOptions);

            StackExchange.using("externalEditor", function() {
            // Have to fire editor after snippets, if snippets enabled
            if (StackExchange.settings.snippets.snippetsEnabled) {
            StackExchange.using("snippets", function() {
            createEditor();
            });
            }
            else {
            createEditor();
            }
            });

            function createEditor() {
            StackExchange.prepareEditor({
            heartbeatType: 'answer',
            autoActivateHeartbeat: false,
            convertImagesToLinks: true,
            noModals: true,
            showLowRepImageUploadWarning: true,
            reputationToPostImages: 10,
            bindNavPrevention: true,
            postfix: "",
            imageUploader: {
            brandingHtml: "Powered by u003ca class="icon-imgur-white" href="https://imgur.com/"u003eu003c/au003e",
            contentPolicyHtml: "User contributions licensed under u003ca href="https://creativecommons.org/licenses/by-sa/3.0/"u003ecc by-sa 3.0 with attribution requiredu003c/au003e u003ca href="https://stackoverflow.com/legal/content-policy"u003e(content policy)u003c/au003e",
            allowUrls: true
            },
            noCode: true, onDemand: true,
            discardSelector: ".discard-answer"
            ,immediatelyShowMarkdownHelp:true
            });


            }
            });














            draft saved

            draft discarded


















            StackExchange.ready(
            function () {
            StackExchange.openid.initPostLogin('.new-post-login', 'https%3a%2f%2fmath.stackexchange.com%2fquestions%2f3014786%2fsmallest-value-of-a2-b2-c2-d2-given-values-for-abcd-ac%23new-answer', 'question_page');
            }
            );

            Post as a guest















            Required, but never shown

























            2 Answers
            2






            active

            oldest

            votes








            2 Answers
            2






            active

            oldest

            votes









            active

            oldest

            votes






            active

            oldest

            votes









            3














            Let $lambda = a + b + c + d$. When you sum over the 3 equations.



            $$(a+b)(c+d)=143,quad (a+c)(b+d)=150,quad (a+d)(b+c)=169tag{*1}$$



            LHS sums to
            $$2(ab+ac+ad+bc+bd+cd) = (a+b+c+d)^2 - (a^2+b^2+c^2+d^2)$$
            while RHS sums to $462$. This leads to
            $$a^2 + b^2 + c^2 + d^2 = lambda^2 - 462$$
            To minimize $a^2+b^2+c^2+d^2$, one just need to minimize $lambda^2$. If you look at LHS
            of the 3 equations, all of them is a product of $2$ factors which sum to $lambda$.
            In general, if we have $p + q = lambda$, then
            $$4pq = (p+q)^2 - (p-q)^2 le (p+q)^2 = lambda^2$$
            The set of 3 equations tell us



            $$begin{align}lambda^2 &ge 4max{ (a+b)(c+d), (a+c)(b+d), (a+d)(b+c) }\
            &= 4max{ 143, 150, 169 }\&= 676end{align}$$



            As a result,
            $$a^2 + b^2 + c^2 + d^2 = lambda^2 - 462 ge 214$$



            To see $214$ is the actual minimum, we need to find $(a,b,c,d)$ which satisfies $(*1)$ and $lambda^2 = 676 = 26^2$.



            Flipping all the signs of $a,b,c,d$ if necessary, we can assume $lambda = 26$.



            The third equation $(a+d)(b+c) = 169 = 13^2 = frac14 (26)^2$ tell us
            $a + d = b + c$.

            Introduce $u,v$ such that



            $$(a,b,c,d) = left( frac{13+u}{2}, frac{13+v}{2}, frac{13-v}{2}, frac{13-u}{2}right)$$



            and substitute into the first and second equation and simplify, we obtain



            $$left(frac{u+v}{2}right)^2 = 26quadtext{ and }quad left(frac{u-v}{2}right)^2 = 19$$



            Using this, we find following 4-tuple



            $$(a,b,c,d) = {small left(
            frac{13+sqrt{26}+sqrt{19}}{2},
            frac{13+sqrt{26}-sqrt{19}}{2},
            frac{13-sqrt{26}+sqrt{19}}{2},
            frac{13-sqrt{26}-sqrt{19}}{2}
            right)}$$

            is a solution of the 3 equations in $(*1)$ with $a + b + c + d = 26$.
            One can verify $a^2 + b^2 + c^2 + d^2 = 214$ for this particular solution. As a result, the lower bound $214$ is achievable and $214$ is the smallest possible value we seek.






            share|cite|improve this answer




























              3














              Let $lambda = a + b + c + d$. When you sum over the 3 equations.



              $$(a+b)(c+d)=143,quad (a+c)(b+d)=150,quad (a+d)(b+c)=169tag{*1}$$



              LHS sums to
              $$2(ab+ac+ad+bc+bd+cd) = (a+b+c+d)^2 - (a^2+b^2+c^2+d^2)$$
              while RHS sums to $462$. This leads to
              $$a^2 + b^2 + c^2 + d^2 = lambda^2 - 462$$
              To minimize $a^2+b^2+c^2+d^2$, one just need to minimize $lambda^2$. If you look at LHS
              of the 3 equations, all of them is a product of $2$ factors which sum to $lambda$.
              In general, if we have $p + q = lambda$, then
              $$4pq = (p+q)^2 - (p-q)^2 le (p+q)^2 = lambda^2$$
              The set of 3 equations tell us



              $$begin{align}lambda^2 &ge 4max{ (a+b)(c+d), (a+c)(b+d), (a+d)(b+c) }\
              &= 4max{ 143, 150, 169 }\&= 676end{align}$$



              As a result,
              $$a^2 + b^2 + c^2 + d^2 = lambda^2 - 462 ge 214$$



              To see $214$ is the actual minimum, we need to find $(a,b,c,d)$ which satisfies $(*1)$ and $lambda^2 = 676 = 26^2$.



              Flipping all the signs of $a,b,c,d$ if necessary, we can assume $lambda = 26$.



              The third equation $(a+d)(b+c) = 169 = 13^2 = frac14 (26)^2$ tell us
              $a + d = b + c$.

              Introduce $u,v$ such that



              $$(a,b,c,d) = left( frac{13+u}{2}, frac{13+v}{2}, frac{13-v}{2}, frac{13-u}{2}right)$$



              and substitute into the first and second equation and simplify, we obtain



              $$left(frac{u+v}{2}right)^2 = 26quadtext{ and }quad left(frac{u-v}{2}right)^2 = 19$$



              Using this, we find following 4-tuple



              $$(a,b,c,d) = {small left(
              frac{13+sqrt{26}+sqrt{19}}{2},
              frac{13+sqrt{26}-sqrt{19}}{2},
              frac{13-sqrt{26}+sqrt{19}}{2},
              frac{13-sqrt{26}-sqrt{19}}{2}
              right)}$$

              is a solution of the 3 equations in $(*1)$ with $a + b + c + d = 26$.
              One can verify $a^2 + b^2 + c^2 + d^2 = 214$ for this particular solution. As a result, the lower bound $214$ is achievable and $214$ is the smallest possible value we seek.






              share|cite|improve this answer


























                3












                3








                3






                Let $lambda = a + b + c + d$. When you sum over the 3 equations.



                $$(a+b)(c+d)=143,quad (a+c)(b+d)=150,quad (a+d)(b+c)=169tag{*1}$$



                LHS sums to
                $$2(ab+ac+ad+bc+bd+cd) = (a+b+c+d)^2 - (a^2+b^2+c^2+d^2)$$
                while RHS sums to $462$. This leads to
                $$a^2 + b^2 + c^2 + d^2 = lambda^2 - 462$$
                To minimize $a^2+b^2+c^2+d^2$, one just need to minimize $lambda^2$. If you look at LHS
                of the 3 equations, all of them is a product of $2$ factors which sum to $lambda$.
                In general, if we have $p + q = lambda$, then
                $$4pq = (p+q)^2 - (p-q)^2 le (p+q)^2 = lambda^2$$
                The set of 3 equations tell us



                $$begin{align}lambda^2 &ge 4max{ (a+b)(c+d), (a+c)(b+d), (a+d)(b+c) }\
                &= 4max{ 143, 150, 169 }\&= 676end{align}$$



                As a result,
                $$a^2 + b^2 + c^2 + d^2 = lambda^2 - 462 ge 214$$



                To see $214$ is the actual minimum, we need to find $(a,b,c,d)$ which satisfies $(*1)$ and $lambda^2 = 676 = 26^2$.



                Flipping all the signs of $a,b,c,d$ if necessary, we can assume $lambda = 26$.



                The third equation $(a+d)(b+c) = 169 = 13^2 = frac14 (26)^2$ tell us
                $a + d = b + c$.

                Introduce $u,v$ such that



                $$(a,b,c,d) = left( frac{13+u}{2}, frac{13+v}{2}, frac{13-v}{2}, frac{13-u}{2}right)$$



                and substitute into the first and second equation and simplify, we obtain



                $$left(frac{u+v}{2}right)^2 = 26quadtext{ and }quad left(frac{u-v}{2}right)^2 = 19$$



                Using this, we find following 4-tuple



                $$(a,b,c,d) = {small left(
                frac{13+sqrt{26}+sqrt{19}}{2},
                frac{13+sqrt{26}-sqrt{19}}{2},
                frac{13-sqrt{26}+sqrt{19}}{2},
                frac{13-sqrt{26}-sqrt{19}}{2}
                right)}$$

                is a solution of the 3 equations in $(*1)$ with $a + b + c + d = 26$.
                One can verify $a^2 + b^2 + c^2 + d^2 = 214$ for this particular solution. As a result, the lower bound $214$ is achievable and $214$ is the smallest possible value we seek.






                share|cite|improve this answer














                Let $lambda = a + b + c + d$. When you sum over the 3 equations.



                $$(a+b)(c+d)=143,quad (a+c)(b+d)=150,quad (a+d)(b+c)=169tag{*1}$$



                LHS sums to
                $$2(ab+ac+ad+bc+bd+cd) = (a+b+c+d)^2 - (a^2+b^2+c^2+d^2)$$
                while RHS sums to $462$. This leads to
                $$a^2 + b^2 + c^2 + d^2 = lambda^2 - 462$$
                To minimize $a^2+b^2+c^2+d^2$, one just need to minimize $lambda^2$. If you look at LHS
                of the 3 equations, all of them is a product of $2$ factors which sum to $lambda$.
                In general, if we have $p + q = lambda$, then
                $$4pq = (p+q)^2 - (p-q)^2 le (p+q)^2 = lambda^2$$
                The set of 3 equations tell us



                $$begin{align}lambda^2 &ge 4max{ (a+b)(c+d), (a+c)(b+d), (a+d)(b+c) }\
                &= 4max{ 143, 150, 169 }\&= 676end{align}$$



                As a result,
                $$a^2 + b^2 + c^2 + d^2 = lambda^2 - 462 ge 214$$



                To see $214$ is the actual minimum, we need to find $(a,b,c,d)$ which satisfies $(*1)$ and $lambda^2 = 676 = 26^2$.



                Flipping all the signs of $a,b,c,d$ if necessary, we can assume $lambda = 26$.



                The third equation $(a+d)(b+c) = 169 = 13^2 = frac14 (26)^2$ tell us
                $a + d = b + c$.

                Introduce $u,v$ such that



                $$(a,b,c,d) = left( frac{13+u}{2}, frac{13+v}{2}, frac{13-v}{2}, frac{13-u}{2}right)$$



                and substitute into the first and second equation and simplify, we obtain



                $$left(frac{u+v}{2}right)^2 = 26quadtext{ and }quad left(frac{u-v}{2}right)^2 = 19$$



                Using this, we find following 4-tuple



                $$(a,b,c,d) = {small left(
                frac{13+sqrt{26}+sqrt{19}}{2},
                frac{13+sqrt{26}-sqrt{19}}{2},
                frac{13-sqrt{26}+sqrt{19}}{2},
                frac{13-sqrt{26}-sqrt{19}}{2}
                right)}$$

                is a solution of the 3 equations in $(*1)$ with $a + b + c + d = 26$.
                One can verify $a^2 + b^2 + c^2 + d^2 = 214$ for this particular solution. As a result, the lower bound $214$ is achievable and $214$ is the smallest possible value we seek.







                share|cite|improve this answer














                share|cite|improve this answer



                share|cite|improve this answer








                edited Nov 27 '18 at 4:26

























                answered Nov 26 '18 at 21:56









                achille hui

                95.6k5130257




                95.6k5130257























                    1














                    Using A.M.-G.M. we get: $$169+143+150=(a+b)(c+d)+(a+c)(b+d)+(a+d)(b+c)=$$



                    $$2ac+2ad+2bc+2bd+2ab+2cd leq 3a^2+3b^2+3c^2+3d^2$$



                    So $$a^2+b^2+c^2+d^2geq 154$$



                    But, unfortenuly this is not the minumum, since variables can not be all the same.






                    share|cite|improve this answer























                    • Cant understand why there's an inquality there
                      – Ahmed I. Elsayed
                      Nov 26 '18 at 21:19










                    • $2xyleq x^2+y^2$........
                      – greedoid
                      Nov 26 '18 at 21:20










                    • 2xy <= x^2 + y^^2 ? not xy <= y^2 + x^2 ?
                      – Ahmed I. Elsayed
                      Nov 26 '18 at 21:24










                    • First.............
                      – greedoid
                      Nov 26 '18 at 21:25










                    • I don't know how your answer addresses the question. You just show a lower bound, not the minimum.
                      – Le Anh Dung
                      Nov 27 '18 at 4:39


















                    1














                    Using A.M.-G.M. we get: $$169+143+150=(a+b)(c+d)+(a+c)(b+d)+(a+d)(b+c)=$$



                    $$2ac+2ad+2bc+2bd+2ab+2cd leq 3a^2+3b^2+3c^2+3d^2$$



                    So $$a^2+b^2+c^2+d^2geq 154$$



                    But, unfortenuly this is not the minumum, since variables can not be all the same.






                    share|cite|improve this answer























                    • Cant understand why there's an inquality there
                      – Ahmed I. Elsayed
                      Nov 26 '18 at 21:19










                    • $2xyleq x^2+y^2$........
                      – greedoid
                      Nov 26 '18 at 21:20










                    • 2xy <= x^2 + y^^2 ? not xy <= y^2 + x^2 ?
                      – Ahmed I. Elsayed
                      Nov 26 '18 at 21:24










                    • First.............
                      – greedoid
                      Nov 26 '18 at 21:25










                    • I don't know how your answer addresses the question. You just show a lower bound, not the minimum.
                      – Le Anh Dung
                      Nov 27 '18 at 4:39
















                    1












                    1








                    1






                    Using A.M.-G.M. we get: $$169+143+150=(a+b)(c+d)+(a+c)(b+d)+(a+d)(b+c)=$$



                    $$2ac+2ad+2bc+2bd+2ab+2cd leq 3a^2+3b^2+3c^2+3d^2$$



                    So $$a^2+b^2+c^2+d^2geq 154$$



                    But, unfortenuly this is not the minumum, since variables can not be all the same.






                    share|cite|improve this answer














                    Using A.M.-G.M. we get: $$169+143+150=(a+b)(c+d)+(a+c)(b+d)+(a+d)(b+c)=$$



                    $$2ac+2ad+2bc+2bd+2ab+2cd leq 3a^2+3b^2+3c^2+3d^2$$



                    So $$a^2+b^2+c^2+d^2geq 154$$



                    But, unfortenuly this is not the minumum, since variables can not be all the same.







                    share|cite|improve this answer














                    share|cite|improve this answer



                    share|cite|improve this answer








                    edited Nov 26 '18 at 20:30

























                    answered Nov 26 '18 at 20:23









                    greedoid

                    38.2k114797




                    38.2k114797












                    • Cant understand why there's an inquality there
                      – Ahmed I. Elsayed
                      Nov 26 '18 at 21:19










                    • $2xyleq x^2+y^2$........
                      – greedoid
                      Nov 26 '18 at 21:20










                    • 2xy <= x^2 + y^^2 ? not xy <= y^2 + x^2 ?
                      – Ahmed I. Elsayed
                      Nov 26 '18 at 21:24










                    • First.............
                      – greedoid
                      Nov 26 '18 at 21:25










                    • I don't know how your answer addresses the question. You just show a lower bound, not the minimum.
                      – Le Anh Dung
                      Nov 27 '18 at 4:39




















                    • Cant understand why there's an inquality there
                      – Ahmed I. Elsayed
                      Nov 26 '18 at 21:19










                    • $2xyleq x^2+y^2$........
                      – greedoid
                      Nov 26 '18 at 21:20










                    • 2xy <= x^2 + y^^2 ? not xy <= y^2 + x^2 ?
                      – Ahmed I. Elsayed
                      Nov 26 '18 at 21:24










                    • First.............
                      – greedoid
                      Nov 26 '18 at 21:25










                    • I don't know how your answer addresses the question. You just show a lower bound, not the minimum.
                      – Le Anh Dung
                      Nov 27 '18 at 4:39


















                    Cant understand why there's an inquality there
                    – Ahmed I. Elsayed
                    Nov 26 '18 at 21:19




                    Cant understand why there's an inquality there
                    – Ahmed I. Elsayed
                    Nov 26 '18 at 21:19












                    $2xyleq x^2+y^2$........
                    – greedoid
                    Nov 26 '18 at 21:20




                    $2xyleq x^2+y^2$........
                    – greedoid
                    Nov 26 '18 at 21:20












                    2xy <= x^2 + y^^2 ? not xy <= y^2 + x^2 ?
                    – Ahmed I. Elsayed
                    Nov 26 '18 at 21:24




                    2xy <= x^2 + y^^2 ? not xy <= y^2 + x^2 ?
                    – Ahmed I. Elsayed
                    Nov 26 '18 at 21:24












                    First.............
                    – greedoid
                    Nov 26 '18 at 21:25




                    First.............
                    – greedoid
                    Nov 26 '18 at 21:25












                    I don't know how your answer addresses the question. You just show a lower bound, not the minimum.
                    – Le Anh Dung
                    Nov 27 '18 at 4:39






                    I don't know how your answer addresses the question. You just show a lower bound, not the minimum.
                    – Le Anh Dung
                    Nov 27 '18 at 4:39




















                    draft saved

                    draft discarded




















































                    Thanks for contributing an answer to Mathematics Stack Exchange!


                    • Please be sure to answer the question. Provide details and share your research!

                    But avoid



                    • Asking for help, clarification, or responding to other answers.

                    • Making statements based on opinion; back them up with references or personal experience.


                    Use MathJax to format equations. MathJax reference.


                    To learn more, see our tips on writing great answers.





                    Some of your past answers have not been well-received, and you're in danger of being blocked from answering.


                    Please pay close attention to the following guidance:


                    • Please be sure to answer the question. Provide details and share your research!

                    But avoid



                    • Asking for help, clarification, or responding to other answers.

                    • Making statements based on opinion; back them up with references or personal experience.


                    To learn more, see our tips on writing great answers.




                    draft saved


                    draft discarded














                    StackExchange.ready(
                    function () {
                    StackExchange.openid.initPostLogin('.new-post-login', 'https%3a%2f%2fmath.stackexchange.com%2fquestions%2f3014786%2fsmallest-value-of-a2-b2-c2-d2-given-values-for-abcd-ac%23new-answer', 'question_page');
                    }
                    );

                    Post as a guest















                    Required, but never shown





















































                    Required, but never shown














                    Required, but never shown












                    Required, but never shown







                    Required, but never shown

































                    Required, but never shown














                    Required, but never shown












                    Required, but never shown







                    Required, but never shown







                    Popular posts from this blog

                    Plaza Victoria

                    In PowerPoint, is there a keyboard shortcut for bulleted / numbered list?

                    How to put 3 figures in Latex with 2 figures side by side and 1 below these side by side images but in...